2
\ begingroup美元

浅水方程是:

\开始{对齐}& \压裂{du} {dt} = - g \压裂{\偏h} {x} \部分+阵线\ \ & \压裂{dv} {dt} = - g \压裂{\偏h}{\偏y} - fu \ \ & \压裂{dh} {dt} = - h(\压裂{\偏u} {x} \部分+ \压裂{\部分v}{\偏y}) \{对齐}结束其中h是浅水的高度(或深度),你美元向东是速度,五美元向北是速度,g是重力加速度和f是科氏参数。

我的问题是如何使线性化方程。从线性化定理,我知道我应该采取{对齐}u = \ \开始眉题{你}+ u ' \ \增加了= \眉题{v} + v”\ \ h = \眉题{h} + h ' \{对齐}结束然后带他们到原始方程,但我就是不知道。从我的理解,把方程h美元作为一个例子,结果应该是{方程}\ \开始压裂{\部分}{\部分t}(\眉题{h} + h) +(\眉题{你}+ u ') \压裂{\部分}{x} \部分(\眉题{h} + h) +(\眉题{v} + v”) \压裂{\部分}{\偏y}(\眉题{h} + h ') = -(\眉题{h} + h)(\压裂{\部分\眉题{你}}{x} \部分+ \压裂{\部分\眉题{v}}{\偏y} + \压裂{\偏u '} {x} \部分+ \压裂{\部分v '}{\偏y}) \{方程}结束通过使用方程{对齐}\ \开始压裂{d \眉题{h}} {dt} = - \眉题{h}(\压裂{\部分\眉题{你}}{x} \部分+ \压裂{\部分\眉题{v}}{\偏y}) \{对齐}结束而忽视条款美元u ' \压裂{\偏h '} {x} \部分$$ v ' \压裂{\偏h '}{\偏y} $,结果我获得{方程}\压裂{\ \开始偏h '}{\部分t} + \眉题{你}\压裂{\偏h '} {x} \部分+ \眉题{v} \压裂{\偏h '}{\偏y} = - \眉题{h}(\压裂{\偏u '} {x} \部分+ \压裂{\部分v '}{\偏y})——h(\压裂{\部分\眉题{你}}{x} \部分+ \压裂{\部分\眉题{v}}{\偏y}) \{方程}结束但在我的教科书,结果是公正的\开始{方程}\压裂{\偏h '}{\部分t} = - \眉题{h}(\压裂{\偏u '} {x} \部分+ \压裂{\部分v '}{\偏y}) \{方程}结束所以我想问,我哪里犯错误吗?实际上,我有同样的问题的线性化方程你美元五美元

\ endgroup美元
1
  • \ begingroup美元 试一试,看看你是否能重新部署额外的术语到额外的术语在v和u-equations,添加后方程的额外条款应该加起来等于零。 \ endgroup美元 2021年8月16日21:22

1回答1

1
\ begingroup美元

好吧,我可以看到一件事。注意,您可以重新排列你导出的方程如下:$ $ \压裂{\偏h '}{\部分t} + \酒吧{你}\压裂{\偏h '} {x} \部分+ \酒吧{v} \压裂{\偏h '}{\偏y} + h \压裂{\部分\酒吧{你}}{x} \部分+ h ' \压裂{\部分\酒吧{v}}{\偏y} = - \酒吧{h} \离开(\压裂{\偏u '} {x} \部分+ \压裂{\部分v '}{\偏y} \右)$ $。观察到左边可以组合在一起使用乘积法则。$ $ \压裂{\偏h '}{\部分t} + \压裂{\部分\酒吧{你}h '} {x} \部分+ \压裂{\部分\酒吧{v} h '}{\偏y} = - \酒吧{h} \离开(\压裂{\偏u '} {x} \部分+ \压裂{\部分v '}{\偏y} \右)$ $

现在有两个假设可以(我没有靠近我在我写这篇文章)。我建议你读的是什么在你的教科书。可能是aomething的滞弹的假设,可能是没有平均流量,等等。然而,假设应该屈服${你}\压裂{\部分\酒吧h '} {x} \部分+ \压裂{\部分\酒吧{v} h '}{\偏y} \ approx0 $${你}\压裂{\部分\酒吧h '} {x} \部分+ \压裂{\部分\酒吧{v} h '}{\偏y} = 0美元

\ endgroup美元
4
  • \ begingroup美元 我明白你的意思,但是我找不到任何假设告诉我我可以带美元\压裂{\部分\眉题{你}h '} {x} \部分+ \压裂{\部分\眉题{v} h '}{\偏y} = 0美元,你能告诉我是什么意思没有平均流量? \ endgroup美元
    - - - - - -
    2021年8月15日4:06
  • \ begingroup美元 哦,我知道我犯了一个错误,我应该把美元\眉题{你}= 0 $和$ \眉题{v} = 0美元,因为它假设流体的基本状态是静止的。 \ endgroup美元
    - - - - - -
    2021年8月15日(美
  • 1
    \ begingroup美元 @Hou,是不正确的。你不要认为液体的基状态为零但一个常数。 \ endgroup美元
    - - - - - -gansub
    2021年8月16日,在47
  • \ begingroup美元 @gansub是的!现在我发现问题! \ endgroup美元
    - - - - - -
    2021年8月16日,在49

你的答案

通过点击“发布你的答案”,你同意我们服务条款并承认您已阅读并理解我们的隐私政策的行为准则

不是你要找的答案?浏览其他问题标记问你自己的问题